Warum brauchen wir in Quantenfeldtheorien eine Randbedingung?

Wenn wir die auf Mannigfaltigkeiten mit einer Grenze definierte Quantenfeldtheorie diskutieren, wählen wir immer eine Randbedingung für die Felder. Und das Argument besagt normalerweise, dass wir die Randbedingung brauchen, um ein wohldefiniertes Variationsproblem zu haben .

Was wir jedoch eigentlich wollen, ist ein wohldefinierter Hilbert-Raum . Wie können wir die Verbindung zwischen diesen beiden scheinbar unterschiedlichen Dingen erkennen?

Die obige Frage ist sinnvoll für Quantenfeldtheorien, die durch eine klassische Aktion definiert sind. Wenn wir allgemeinere Quantenfeldtheorien betrachten, ohne auf die Wirkung Bezug zu nehmen, warum brauchen wir dann eine Randbedingung, damit die Theorie wohldefiniert ist?

Vielleicht hilft meine Antwort hier auf eine andere "Warum" -Frage physical.stackexchange.com/questions/230703/… . Warum Fragen zu grundlegenden axiomatischen Annahmen führen, die durch die Daten auferlegt werden, und die Notwendigkeit der Vorhersagbarkeit für neue Daten, damit eine Theorie der Physik validiert wird.
Ihre Frage sollte von einem Theoretiker beantwortet werden, da es um die Vereinbarkeit zweier mathematischer Formulierungen für eine Feldtheorie geht. Ich füge nur hinzu, dass Randbedingungen von grundlegender Bedeutung sind, um aus der Fülle mathematischer Anordnungen die Teilmenge auszuwählen, die zu physikalischen Daten passt, und die für die Validierung wichtig sind. auch Vorhersage neuer Daten.

Antworten (1)

Nun, wenn Sie an QFT in seiner allgemeinsten Formulierung denken wollen – ich würde sagen, dass es nicht so sehr darauf ankommt, dass Sie Randbedingungen wählen müssen , sondern dass Sie können .

Lassen H B sei der einer Randbedingung zugeordnete Hilbert-Raum B . Wenn Sie keine Randbedingungen aufstellen möchten, können Sie stattdessen erklären, dass Ihr Hilbert-Raum ist

H = B H B
wobei die Summe über alle zulässigen Randbedingungen geht, an denen Sie interessiert sein könnten. (Wenn Sie es beispielsweise mit einer CFT zu tun haben, möchten Sie möglicherweise nur über bcs summieren, die nur konform sind usw.)

Technisch gesehen müssen Sie also keine Randbedingung angeben. Du könntest gehen B willkürlich, indem einfach alle Randbedingungen zugelassen werden.

Der interessante Aspekt ist jedoch, dass Sie eine Randbedingung auswählen können . Anstatt mit dem absurd großen Hilbertraum zu arbeiten H , können Sie durchgängig einen kleineren Abstand auswählen H B . Dies ist im Allgemeinen nicht möglich: Wenn Sie mit einem Hilbert-Raum beginnen, können Sie normalerweise keinen kleineren Raum auswählen und trotzdem eine konsistente Theorie erhalten. Ein Unterraum einer Theorie ergibt normalerweise keine gute Theorie: Sie können die Einheitlichkeit/Lokalität/Kausalität verlieren, der Unterraum behält möglicherweise nicht alle Symmetrien bei, die Sie benötigen (z. B. Poincaré) usw. Die Behauptung lautet: für die Grenze Bedingungen, der Raum H B H gibt Ihnen eine gute Theorie von selbst.

In diesem Sinne können Sie sich Randbedingungen als "nicht reduzierbare Komponenten" einer QFT vorstellen, ähnlich wie irreduzible Darstellungen allgemeine Darstellungen tun. Wiederholungen müssen nicht irreduzibel sein, aber es ist sinnvoll, mit Irreps zu arbeiten, da alle anderen als Summe dieser Wiederholungen geschrieben werden können. Und Irreps können nicht weiter unterteilt werden, also sind sie minimal. Ebenso können Sie bcs unspezifiziert lassen, aber es ist nützlich, mit bestimmten bcs zu arbeiten, da QFTs ohne bcs als eine Sammlung von QFTs mit bcs betrachtet werden können.

Danke für die nette Antwort! Darf ich eine Anschlussfrage zu den Themen zu (Eich-)Symmetrien stellen? Um konkret zu werden, nehmen Sie die 1+1d reine Maxwell-Theorie auf einem Linienintervall. Unterschiedliche Auswahlen von Randbedingungen können die ursprüngliche Spursymmetrie unterschiedlich brechen. Es ist ein bisschen seltsam, diese zu kombinieren H B durch eine direkte Summe, da jeder von ihnen eine andere Eichsymmetrie respektiert. Wie soll ich das verstehen?
Ich denke, du wolltest eine direkte Summe schreiben , nicht das Tensorprodukt .